LSAT and Law School Admissions Forum

Get expert LSAT preparation and law school admissions advice from PowerScore Test Preparation.

User avatar
 Dave Killoran
PowerScore Staff
  • PowerScore Staff
  • Posts: 5862
  • Joined: Mar 25, 2011
|
#78448
Complete Question Explanation
(The complete setup for this game can be found here: lsat/viewtopic.php?t=15592)

The correct answer choice is (C)

The question stem suspends the fourth rule and replaces it with a rule that states that the computers in offices 2 and 3 were purchased in the same year. Because all of the other conditions remain the same, the only year that the two computers could have been purchased in is 1988 (this occurs because of the interaction of the fifth rule, first rule, and second rule). If the computers in offices 2 and 3 were purchased in 1988, then from the second and third rules, the printers in offices 1 and 4 must have been purchased in 1988. The only uncertainty, then, is whether the printer in office 2 was purchased in 1988 or 1989, and whether the computer in office 4 was purchased in 1987 or 1988:
PT1-Jun1991 game 2 #13 diagram 1.png
Hence, only the printer in office 2 could have been purchased in 1989, and answer choice (C) is correct.
You do not have the required permissions to view the files attached to this post.

Get the most out of your LSAT Prep Plus subscription.

Analyze and track your performance with our Testing and Analytics Package.